LSAT and Law School Admissions Forum

Get expert LSAT preparation and law school admissions advice from PowerScore Test Preparation.

User avatar
 Dave Killoran
PowerScore Staff
  • PowerScore Staff
  • Posts: 5850
  • Joined: Mar 25, 2011
|
#88556
Complete Question Explanation
(The complete setup for this game can be found here: lsat/viewtopic.php?f=182&p=88548#p88548)

The correct answer choice is (E).

If M’s delivery is first and L’s delivery is third, the following initial diagram results:

G4-Q20-d1.png

This scenario can only occur under Template #1. In that template, F and H are delivered earlier than G :longline: K. Thus, with only four available spaces, F and H must be delivered second and fourth (not necessarily in that order), and G and K must be delivered fifth and sixth:

G4-Q20-d2.png

Regardless of the placement of F and H, the third rule will satisfied.

Accordingly, answer choice (E) must be true and is correct.
 paullmatt
  • Posts: 2
  • Joined: Sep 05, 2013
|
#10780
September 2007 LSAT, Section 2, Question 20

For the life of me I cannot figure out where I went wrong. I marked B but the correct answer is being shown as E.

It appears to me that both B & E Must Be True. Any assistance solving this would be much appreciated.
User avatar
 Dave Killoran
PowerScore Staff
  • PowerScore Staff
  • Posts: 5850
  • Joined: Mar 25, 2011
|
#10781
Hi Paul,

Thanks for the question. H's delivery could be second or fourth, and thus while (B) could be true, it does not have to be true. Here's what happens:

If M’s delivery is first and L’s delivery is third, the following initial diagram results:


..... ..... M _ L _ _ _
..... ..... 1 2 3 4 5 6


This scenario can only occur under Template #1 of the game (where M > H). In that template, F and H are delivered earlier than G > K. Thus, with only four available spaces, F and H must be delivered second and fourth (not necessarily in that order), and G and K must be delivered fifth and sixth:


..... ..... M F/H L H/F G K
..... ..... 1 2 3 4 5 6


Regardless of the placement of F and H, the third rule will satisfied.

Accordingly, answer choice (E) must be true and is correct.
 paullmatt
  • Posts: 2
  • Joined: Sep 05, 2013
|
#10802
Thank you for the clarification. My mistake was that I was taking the contrapositive of if F>M ---> L>H to mean that L could not come before H at all...

Get the most out of your LSAT Prep Plus subscription.

Analyze and track your performance with our Testing and Analytics Package.